« first day (42 days earlier)      last day (532 days later) » 
00:00 - 16:0016:00 - 00:00

12:22 AM
0
Q: Step-by-step integration of e^-xy*sin(x)

jdash Integrate \int (e^-xy)*sin(x) step-by-step Found solution for e^x*sin(x). It used integration by parts. Tried it here but keep getting too many Ys.

Welcome to Math.SE, jdash. This site uses MathJax formatting of formulas. More tips here. (autocomment)Normal Human 21 secs ago
0
Q: Natural Cubic Spline Problem - what do I do next?

Jessy CatRelated to this similar problem. I am attempting to solve the following: Find a natural cubic spline function whose knots are $-1$, $0$, and $1$ and that takes on the values $S(-1)=5$, $S(0)=7$, and $S(1)=9$. Neither my textbook nor my class notes have been particularly helpful in showing ...

0
Q: Basis for $L_p$ space

user2770287I am trying to implement an algorithm described in a paper. At one point, it says "Let $\phi_0,\phi_1,\ldots$ be a collection of functions from $\mathbb{R}^d$ to $\mathbb{R}$ such that $\phi_0\equiv1$ and $\{\phi_i(X_t)\}$ form a basis for $L^2(\Omega,\sigma(X_t),\mathbb{Q})$ for all $t=1,\ldot...

0
Q: Basic Combinatorics: How many sequences have at least 3 red balls?

Hani Al-shafeiPick a sequence of 10 balls from a sack containing red, blue, green, yellow, white and black balls. Each time a ball is picked, it is replaced in the sack before the next ball is picked. a) How many sequences have exactly 3 balls? b) At least 4 blue balls? c) exactly 2 black balls AND at ...

0
Q: Square of an odd integer is odd, square of even integer is even, what is the case for higher powers?

grayQuantAre there rules for higher powers? It seems like even and odd is preserved by powers, but how do I prove that?

0
Q: P(Min of 3 Dice Rolls < 3)

Linsu HanI'm trying to solve this with two methods. P(Min of 3 Dice Rolls < 3) = P(D1 < 3 U D2 < 3 U D3 < 3) = 1 - P(D1 >= 3 n D2 >=3 n D3 >=3) For the union probability I'm getting .7 repeating, while for the intersection probability I'm getting .703 repeating. Can someone show me both methods are ...

0
Q: Show that the set of polynomials with rational coefficients is countable.

MatthewProblem: Show that the set of polynomials with rational coefficients is countable. Idea: We know that the set of rational numbers is denumerable. This implies that the set of rational numbers is countable. We also know that the degree that each polynomial can be is a natural number (I think, an...

0
Q: Cyclic Quadrilateral

samsoniteIf i'm given 3 distinct points (a, b, and c) how would i describe the set of all points (d) in which abcd is a cyclic quadrilateral? I'm thinking that it is any point on the circle which contains triangle abc with exception of the vertices of said triangle. Is this correct?

Short title. Cyclic Quadrilateral
0
Q: Proof Suppose that there is a function mapping S onto T. Show that Card(S) is greater than or equal to Card(T)

MatthewProblem: Suppose that there is a function mapping S onto T. Prove that Card(S) is greater than or equal to Card(T). Issue: I can't seem to find a reason why this follows. If S is onto T then this guarantees that for any element in T there is at least one element in S. Therefore, I keep coming u...

0
Q: Calculus word problem

LilA 10 foot high tank shaped like an inverted pyramid with a square base measuring 16ft by 16ft is filling with water at a rate of 2.5 cubic feet per a second. How fast is the water rising when the water is 8 feet high.

Short title. Title contains problem. Calculus word problem
0
Q: Calculus optimization problem

LilJohn is building a rectangular pen against his garage using wooden fencing. He wants to divide it further into 4 smaller pens all of equal size by placing 2 perpendicular units of fencing in the middle. John has 100 feet of fencing and wants to make the pen as big as possible. Find both the dimen...

Short title. Title contains problem. Calculus optimization problem
 
12:59 AM
0
Q: Who deleted my comment on this question and why?

Dan DascalescuI posted a comment on this question thinking Programmers.SE would be a better fit for it. Apparently I was wrong, and two other commenters explained why. But my comment was also deleted, and now the other two folks' comments are out of context. Who deleted my comment? Why? Why wasn't I notifie...

 
0
Q: All open subsets of the spectrum of a number field are principal

Alex SaadLet $K$ be a number field, $\mathcal{O}_k$ its ring of integers, $\operatorname{Cl}(K)$ its class group and $h_K = \lvert \operatorname{Cl}(K)\rvert$ its class number. Let $X = \operatorname{Spec}(\mathcal{O}_K)$ and let $U\subseteq X$ be an open subset of $X$. Apparently (ex.3.19 Liu), using the...

0
Q: Prove that if 5n^2 - 3 is odd then n is odd

MitchI tried to prove this by contradiction and what I did is. I used contradiction proof that is n is odd then 5n^2 - 3 is even. But my Professor said this is wrong you need to prove that 5n^2 - 3 is even then n is odd. Can someone help me please?

Welcome to Math.SE, Mitch. Tag (proof-verification) should not be the only tag a question has. Please add a tag for a subject area to which the question belongs. (from a bot)Normal Human 21 secs ago
0
Q: Sample standard deviation and population sd question

FrasierSample deviation is calculated as population standard deviation/sqrt(sample size) σ/√(n). Then why in an example like this below, is the answer 5.086? I understand that it comes from 4.2+1.96*1.5/sqrt(11), but why are we dividing with sqrt(11) when going from sample to population, when according ...

Welcome to Math.SE, Frasier. Words such as question do not add information to titles. Please edit the title so that it better describes the specifics of your question. Do not hesitate to make it longer or include a formula if needed. This site uses MathJax formatting of formulas. More tips here. (autocomment)Normal Human 21 secs ago
0
Q: Find the domain, x intercepts, y intercepts

LilLet f(x)=x^2+1/x^2-9 So to find the y intercept I take f(0) correct? So when I substitute 0 for x I got -1/9 so is the y intercept (0,-1/9) Also to find the x intercept I set the numerator equal to 0. So then I got x^2+1=0 but wouldn't that make x^2=-1 which is imaginary? I'm a little confused ...

This site uses MathJax formatting of formulas. More tips here. (autocomment)Normal Human 21 secs ago
0
Q: Could you suggest books, papers or problems that could be used as good "general" motivating examples of calculus application?

DanielTheRocketManI would like to stress the kind of reference I am looking for... In statistics there are lots of motivating (and sometimes unexpected) examples that are interested for everyone such as Birthday Problem, Simpson Paradox, secretary problem, St Petersburg Paradox that easily motivate people from dif...

0
Q: $\displaystyle\int_{C_N} \dfrac{dz}{z^2\sin(z)}$ complex integral, problem with residues

J.Doe Let be $C_n$ the rectangle, positively oriented, which sides are in the lines $$x=\pm(N+\dfrac{1}{2})\pi~~~y=\pm(N+\dfrac{1}{2})\pi$$ with $N\in\mathbb{N}$. Prove that $$ \displaystyle\int_{C_N} \dfrac{dz}{z^2\sin(z)}=2\pi i\left[ \frac{1}{6}+2\sum_{n=0}^\infty \dfrac{(-1)^n}{n^2\pi^2}\ri...

Tall formulas in titles break the layout of question lists. Please replace \dfrac with \frac, remove \displaystyle in the title. (autocomment)Normal Human 21 secs ago
 
1:18 AM
1
Q: Is voting up a duplicate comment the same thing as marking it as a duplicate?

intboolstringI have noticed now that when you mark a question as a duplicate, a comment is automatically added in the nature of: Possible Duplicate [Question Title of Duplicate Goes Here] I have also noticed that if you go through the flag menu and mark the question as duplicate of the same question the...

 
0
Q: The Schwartz function and the sobolev space $W^{2,p}$

math101How to prove the Schwartz function in $\mathbb{R}^n$ is dense in the space $W^{2,p}(\mathbb{R}^n)?$

0
Q: Contour Integration with cosn

billybobetboHow I calculate this integral using contour integration? $\int_0^\pi \frac{3cos(n\theta)}{5+4cos(n\theta)}d\theta$ I know I can start by using that $cos(n\theta) = Re (e^{in\theta})$, but I get bogged down in the computing.

0
Q: Determining whether a field extension is a Galois Extension

Mark LucasLet E/F be a field extension. (a) If (E : F) = 2, is E/F a Galois extension? Justify your answer. (b) If (E : F) = 3, is E/F a Galois extension? Justify your answer.

0
Q: Rudin: $y-x>0 \rightarrow \exists n \, n(y-x)>1$

YoTengoUnLCDI found the following in Baby Rudin: Theorem If $x,y\in \Bbb R, x>0$ then there exists a natural number $n$ such that $$nx>y$$ After this, he proves another theorem using the following Let $x<y$, we have $y-x>0$ then the theorem above yields that a natural number $n$ exists such ...

0
Q: Prove that for all positive $x,y,z$, $(2e^x+\dfrac{2}{e^x})(2e^y+\dfrac{2}{e^y})(2e^x+\dfrac{2}{e^x}) \geq 64$

JoshuaProve that for all positive $x,y,z$, $(2e^x+\dfrac{2}{e^x})(2e^y+\dfrac{2}{e^y})(2e^z+\dfrac{2}{e^z}) \geq 64$ I dont have that much experience with inequalities but I know I can rewrite $64 $ as $4^3$ So here is my approach, if I can maybe prove that $(2e^x+\dfrac{2}{e^x}) \geq4$ as well for $...

Welcome to Math.SE, Joshua. Tall formulas in titles break the layout of question lists. Please replace \dfrac with \frac in the title. (from a bot)Normal Human 21 secs ago
0
Q: Is a congruent to b modulo 1 for all a, b E Z?

Niki MInstinctively I think yes, since a = k + b for all a and b, but then I think about, eg. 4 congruent to 7 (modulo 1) and I'm confused about what the least residue of that would be...

0
Q: Show that $\sum_{i=1}^{100}(n+i)^{n+i}\equiv 0 mod 100$

user298507I am trying to show that $$\sum_{i=1}^{100}(n+i)^{n+i}\equiv 0 mod 100$$ Any tips?

0
Q: weak derivative of 1/r^n

user298509I don't know weak derivative of 1/r^n where r is norm of x. (i.e. distance of from origin to x) If n=1, it's easy by logarithm and integration by parts because log(x) is locally integrable. but if n>1, how to approach to it?

0
Q: What is the value of this double integral for this function?

user2598152Double Integral Problem I just don't understand how to do the last step and calculate the actual value with the integral with respect to y. Any help is appreciated. Thank you so much!

0
Q: Coin Tosses Conditioning

bcfWhat is the expected number of tosses of fair coin to get $THH$? Here's my approach: Let $N := $ number of tosses to get $THH$. Then by partitioning the sample space, $$ E(N) = E(N; H) + E(N; T). $$ If we get a $H$ on the first toss (with probability $\frac{1}{2}$) then we basically start over...

0
Q: Question on Sturm Liouville boundary value problems

ODE learnerTHis is an exercise on Martin Braun's Differential Equations and Their Applications p.544 Exercise 5. Let $P_n(x)$ be the Legendre polynomial of degree n. (a) Show that $P_n'(x)$ satisfies a selfadjoint equation with $\lambda = n(n+1) - 2$ (b) Show that $\int_{-1}^1 P_n'(x) P_m'(x) (1-x^2)dx ...

Welcome to Math.SE, ODE learner. Words such as question are uninformative in titles. Please edit the title so that it better describes the specifics of your question. Do not hesitate to make it longer or include a formula if needed. More tips here. (from a bot)Normal Human 21 secs ago
0
Q: Injective but not surjective function.

Niki MI'm trying to think of functions that are injective but not surjective. I can think of f(x) = e^x for f: R --> R (since the range should be positive R for this to be surjective, but R+ is a subset of R) and f(a,b) = a^b for f: ZxZ --> R (the range should be Q for this to be surjective, but Q is a...

Questions tend to get more attention when they have a tag for a broad area of mathematics relevant to the question. Some of these tags might fit. (from a bot)Normal Human 21 secs ago
0
Q: Solution Sets of Linear Systems (Very Basic Question)

MuuzA is a 2x5 matrix with two pivot positions. Does the equation Ax=b have at least one solution for every possible b? If the answer is yes, then I understand why. If the answer is no, please explain.

Welcome to Math.SE, Muuz. Words such as question are uninformative in titles. Please edit the title so that it better describes the specifics of your question. Do not hesitate to make it longer or include a formula if needed. More tips here. (autocomment)Normal Human 21 secs ago
0
Q: MGF : Actuarial problem. How to find the requested probability.

Electro82 The problem said: Let X and Y be identically distributed independent random variables such that the moment generating function of X + Y is M(t) = 0.09e 2t + 0.24e t + 0.34 + 0.24et + 0.09e2t, −1 < t < 1 Calculate P(X <=0). I found the path to have the correct answer: We must just b...

This site uses MathJax formatting of formulas. More tips here. (autocomment)Normal Human 21 secs ago
0
Q: An exercise about repeated root

PeterI want to prove the following: Let $K$ be a field, let $g(X) \in K[X]$ be an irreducible polynomial, and let $L$ be a splitting field for $g(X)$ over $K$. Prove that $g(X)$ has at least one repeated root in $L$ if and only if char($K$)$ = p$ for some $p > 0$ and $g(X) = h(X^p)$ for some $h(X) \i...

Title contains exercise. An exercise about repeated root
0
Q: Finding a unique continuous function

user298514Let $f$ be a given continuous function on $[0,1]$. Prove that there is a unique continuous function $g$ on $[0,1]$ satisfying $$g(x) = \frac{1}{2}g\left(\frac{x+1}{2}\right) + f(x)$$ for all $x$ in $[0,1]$.

Welcome to Math.SE, user298514. Consider replacing (analysis) with a more specific tag for the relevant branch of analysis. (from a bot)Normal Human 21 secs ago
0
Q: What is the distribution?

Sung Pang$X_1,X_2,...,X_n\;i.i.d\;$with bernoulli distribution(p.m.f is $\theta^{x}(1-\theta)^{1-x})$. I want to gain distribution of $\bar X$. Then I calculated moment generating function of $\bar X$(by calculating $E[e^{\frac t n (X_1+...+X_n)}]$. And I gained M.G.F of $\bar X$. Result is $(1-\theta...

 
2:44 AM
0
Q: Volume of a paperweight

GuestCalcA paperweight has a slanted top described by x + y + z = 2. Its edges are orthogonal to the xy–plane, and the bottom of the paperweight is formed by the triangle with vertices (1, 0, 0),(0, −1, 0) and (0, 1, 0). Use a triple integral to find the volume of the paperweigh. From the base of the gra...

 
0
Q: I love hats too, but is editing like this acceptable?

Dave ChenI love hats too, but this user loves hats way too much: In the past 10 minutes they edited 5 old posts, probably for a hat :D Each one of these posts only add a few back-ticks. Is this something that these hats are encouraging?

 
3:00 AM
0
Q: Let $R = M_n(k)$, where $k$ is a field. Then any R-module that is finite dimensional over K is a direct sum of

algebraisomorphic copies of $V$, where $V = k^n$. I was able to show that R has a unique simple module V, but I'm stuck on the rest of the exercise. Any help is appreciated.

0
Q: Graph Sketching Understanding

Mone Skratt HenrySketch the graph of the function y=x(4-x)-83lnx. Indicate the transition points (local extrema and points of inflection). Local minimum x= Local maximum x= inflection at x= I understand how to solve this problem mathematically, however, is there any way of analyzing the question to get a better...

0
Q: can you please check my professor's work I think he may be wrong?

AmandaHi I was checking my answers with my professor and I think the way he solved it was wrong can you please check? Question: Let R be the region in the xy-plane bounded by the axes and the line x + y = 1. Use the change-of-variable transformation T(u, v) = (u + v,u v) to evaluate, ZZ R(x y)5(x + y)...

This site uses MathJax formatting of formulas. More tips here. (autocomment)Normal Human 20 secs ago
0
Q: How to show $ \sup\limits_{k}||u_k||_{W^{1.q}(U)}<\infty $?

lanse7ptyIf $u_k\rightharpoonup u$ weakly in $W^{1,q}(U)$,how to show $$ \sup\limits_{k}||u_k||_{W^{1.q}(U)}<\infty $$

Title contains \limits. Short question. How to show $ \sup\limits_{k}||u_k||_{W^{1.q}(U)}<\infty $?
0
Q: Should I simplify the compund function f/g before calculating the domain?

user273872Consider the functions f(x)=(x^2-14x+49) and g(x)=(x-7) and f(x)/g(x)=(x^2-14x+49)/(x-7), notice how f is g^2 because you can factor (x^2-14x+49)=(x-7)^2 so f/g = (x-7)^2/(x-7) = (x-7). So since f/g = (x-7) which is just a line the domain should just be (-infinity, infinity) right? But here is wh...

Welcome to Math.SE, user273872. This site uses MathJax formatting of formulas. More tips here. (autocomment)Normal Human 20 secs ago
0
Q: Localization of a PID is a PID

Ethan AlwaiseI would like a verification of a proof for the following statement. Let $S$ be a multiplicatively closed subset of a ring $R$. If $R$ is a PID, then $S^{-1}R$ is a PID. Let $I = \left<r_1/s_1, r_2/s_2\ldots\right>$ be an ideal of $S^{-1}R$. Since $1/s_i$ is a unit in $S^{-1}R$, we have $\left<r_...

Tagged proof-verification. Localization of a PID is a PID
 
3:28 AM
0
Q: How to evaluate this definite double integral?

d0rmLife$$\int\int_D 6x\sqrt{y^2-x^2}dA, D=\{(x,y)|0\leq y\leq 2, 0 \leq x \leq y\}$$ I tried: $$\int_0^2 \int_0^y 6x\sqrt{y^2-x^2}dxdy$$ But that is incorrect.

0
Q: I have a question, please help me.

Nguyen Nhu QuanLet $Au_i=u_{i+1}-(2-\beta)u_i+u_{i-1}$ whith $u\in \ell^2=\{(u_i)_{i\in Z}:\sum_{i\in Z}u_i<+\infty\}$. How to compute $||A^{-1}||$ or estimate it?? Is it a sectorial operator of type $(\omega, \theta)$ with $\omega <0$????

Welcome to Math.SE, Nguyen Nhu Quan. Words such as question, please, help do not add information to titles. Please edit the title so that it better describes the specifics of your question. Do not hesitate to make it longer or include a formula if needed. More tips here. (from a bot)Normal Human 20 secs ago
0
Q: Inquiry about U-Substitution.

Mone Skratt HenryHow come in u-substitution if u is to the -1 power it becomes equivalent to the ln of u? I don't have a specific problem where that is the case , however I do recall this being a rule of thumb. Can someone explain this to me?

0
Q: Proofs problem with bijection

Eugene WalesLet $f : A \rightarrow B$. Prove that if $X \subseteq A, Y \subseteq B$, and $f$ is a bijection, then $f(X) = Y$ if and only if $f^{-1}(Y) = X$.

Short title. Title contains problem. Short question. Tagged proof-writing. Proofs problem with bijection
0
Q: A query about reducting SAT to 3SAT when there are more than 3 literals in a clause

Chen LuC=a∨b∨c∨d∨e is a clause in SAT D= (a∨b∨x)∧ (¯x∨c∨y)∧ (¯y∨d∨e) is the another form of C to make sure every clause has only threeliterals Is D true when C is true and false when C is false? Why? I think when a,x,y=true and b,c,d,e=false, then C is true but D is false.

0
Q: Does anyone have a list of Mathematics textbooks whose copyright has expired?

user1172468As the title says I'm looking for Math textbooks whose copyright has expired or have an open source / creative commons compatible copyright.

Tag (reference-request) should not be the only tag a question has. Please add a tag for a subject area to which the question belongs. (from a bot)Normal Human 20 secs ago
0
Q: Is there an easy way to show this matrix is totally unimodular?

TurboConsider $$\begin{bmatrix} C_{11}&C_{12}&\dots&C_{1n}\\ C_{21}&C_{22}&\dots&C_{2n}\\ \vdots\\ C_{n1}&C_{n2}&\dots&C_{nn}\\ D_{11}&D_{12}&\dots&D_{1n}\\ D_{21}&D_{22}&\dots&D_{2n}\\ \vdots\\ D_{n1}&D_{n2}&\dots&D_{nn}\\ \end{bmatrix}$$ where each $C_{ij}$ and $D_{ij}$ is $1\times n$ submatrix. $...

0
Q: Tricky Linear Algebra syntax -- help me read it? Pic attached

Asa WittPicture Link Here If someone could please explain in English what the question is actually asking, that'd be great. An explanation would be nice too, but no worries. (I know the answer is e, it's not homework)

Title contains help, tricky. Question contains please. Tricky Linear Algebra syntax -- help me read it? Pic attached
0
Q: Finding the order of pole of $f(z)=\frac{\sin z}{z-\pi}$

Jinmoo YouThe problem is Kreyszig 10ed international edition : 16.2 #9. What is the order of pole at $z=\pi$ of the function $f(z)$ below? $$f(z)=\frac{\sin z}{z-\pi}$$ I thought that it will be simple pole because $\sin z$ is analytic for all $z$$\in$$\Bbb C$ and $z-\pi$ has a 1st order zero at $z=\pi$....

Questions tend to get more attention when they have a tag for a broad area of mathematics relevant to the question. Some of these tags might fit. (from a bot)Normal Human 21 secs ago
0
Q: Defining Compound Function

Martin RandIf f(x) = (x+x) and g(x) = (x-5)*2, and we have the compound function g(f(x)), how can we "define what the resulting function does"? It's obvious what is happening, but I'm not quite sure how this question is asking and how it would be laid out. Perhaps, g(x) = ((x+x)-5)*2

 
4:23 AM
0
Q: how to prove this? unitary matrix representation

Kak Suif $f$ is a unitary matrix representation, then $f$ is a equivalent to a direct sum of irreducible unitary representation via an equivalence implemented by a unitary matrix $T$.

0
Q: Simple summation question regarding origin of summand:

BLAZEIf $$\frac{1}{\sigma_\widehat{e}^2}=\sum_i\frac{1}{\sigma_i^2}\tag{1}$$ Pick anyone of the $\sigma_j$ and multiply both sides of $(1)$ by $\sigma_j^2$ $$\implies\frac{\sigma_j^2}{\sigma_\widehat e^2}=\color{red}{\fbox{$\sum_i\frac{\sigma_j^2}{\sigma_i^2}=\color{blue}{1}+\sum_{i\ne j}\frac{\sigm...

Words such as question do not add information to titles. Please edit the title so that it better describes the specifics of your question. Do not hesitate to make it longer or include a formula if needed. More tips here. (autocomment)Normal Human 21 secs ago
0
Q: I can't understand to solve the question (Khan Academy Algebra Basics - Exponent Properties)

user298530I was learning (practicing to solve) simplifying the rational expressions. I know how to simplify the rational expressions... but I can't understand some part of the questions. The question that I can't understand If you look at this image, you could see sentence "First, let's set the denominat...

0
Q: how to find this function is bijective?

Zeshan MuztarLet A={x∈R, x not equal 1} B={x∈R, x not equal 2} f(x)=2x/(x-1) A -> B How to find the bijective of f(x)?

 
0
Q: Хотфиксы на странице выборов модераторов

Nick VolynkinПока выборы не начались, хорошо бы исправить ошибки на странице выборов. http://ru.stackoverflow.com/election/1 Полужирным выделил необходимые правки. Модераторы нашего сообщества должны: быть терпеливыми и честными подавать личный пример демонстрировать уважение к другим участникам сообщест...

0
Q: Get Secret Hats in winter bash 2015

Creative AndroidHow do I get secret hats in winter bash 2015? I wonder if I could wear as many as hats I get

 
4:47 AM
0
Q: Mathematical Law vs. Axiom

B. Allan What are the differences between a so called mathematical law and axiom? Please define both concepts in your answer.

Tag (definition) should not be the only tag a question has. Please add a tag for a subject area to which the question belongs. (autocomment)Normal Human 21 secs ago
0
Q: using an alternate definition for Exponential and Logarithmic functions ( and by extension sin and cos related ) to prove identities

ArjangAre there better alternative definitions than $\exp(x) = {\large\sum\limits_{k=0}^\infty} \dfrac{x^k}{k!} , \ln(x) = {\large\int_1^x} \dfrac1t\ dt$. that can be used for derivation of their identities e.g. $\exp (x+y)=\exp(x)\exp(y)$, $\ln (xy) =\ln (x)\ln(y)$ ( and by extension sin and cos rela...

Tag (definition) should not be the only tag a question has. Please add a tag for a subject area to which the question belongs. (autocomment)Normal Human 21 secs ago
0
Q: How does the degree of a field extension determine if it is Galois?

YakIf E/F is a field extension and (E:F)=2, is it Galois? What if (E:F)=3? I don't see why the degree of the field extension matter if it is finite. Thanks!

0
Q: Recursive Definition

Martin Rand Consider the following informal definition for a function calc(x,y) = (0*y) + (1*y) + … + (x*y) For example, we have that calc(2,5) = (0*5) + (1*5) + (2*5) Give a recursive definition for the function calc. Give a trace to show each step involved in calculating calc(3,4) using your def...

This site uses MathJax formatting of formulas. More tips here. (autocomment)Normal Human 21 secs ago
 
0
Q: What is 'Hat' on Web Application main site?

serenesatI got a new notification You earned Sufganiyot on Web Application!. What is this Hat? You can see in my profile picture I wore this. Can anyone give a brief information about this?

 
0
Q: How to solve $Ri(t)+ L\frac{di}{dt}= A\cos(\omega t+ \alpha)\;?$

user36790How to solve this differential equation:$$Ri(t)+ L\frac{di}{dt}= A\cos(\omega t+ \alpha)\;?$$ Any hint would be appreciated.

 
5:11 AM
0
Q: A question about KKT condition

addddddcGiven an optimization as follows: \begin{align} \text{minimize}\quad &c^Tx \\ \text{subject to}\quad &Ax = 0 \\ & \|x\|_2^2 \leq 1 \end{align} where $A \in \Re^{m\times n}$ and $c \in \Re^n$. Let $x^*$ be the optimal solution to this problem. Because $c^Tx$, $\|x\|_2^2 - 1$ are convex and...

0
Q: Poincare Triangles given Vertices

samsoniteIf I'm given 3 vertices, say (5,3), (9,3), and (9,7), how can I draw this triangle in a 2 dimensional way along 2 axises and still represent it as a poincare triangle?

Welcome to Math.SE, addddddc. Words such as question are uninformative in titles. Please edit the title so that it better describes the specifics of your question. Do not hesitate to make it longer or include a formula if needed. More tips here. (from a bot)Normal Human 21 secs ago
0
Q: very quick question about linear transformations

user3533755$T\begin{bmatrix}a\\b\\c\\d\end{bmatrix} = 2a-3b+c-2d$ $v_1=\begin{bmatrix}1\\2\\3\\4\end{bmatrix}$ $v_2=\begin{bmatrix}1&2&3&4\end{bmatrix}$ then $T(v_1) = -9$ or $\begin{bmatrix}2\\-6\\3\\-8\end{bmatrix}$? then would $T(v_2) = -9$ or $\begin{bmatrix}2&-6&3&-8\end{bmatrix}$ or what?

Words such as question do not add information to titles. Please edit the title so that it better describes the specifics of your question. Do not hesitate to make it longer or include a formula if needed. More tips here. (autocomment)Normal Human 21 secs ago
0
Q: What are some major open problems in Galois theory?

user290591Few days back one of my friend and I were discussing about Galois life and his ideas. Though we are not trained in Galois theory, but I am recently started to learn it by myself and hope to take up research on it some times soon. So I am wondering about the open problems in Galois theory? Is ther...

Title contains problem. Question contains please. What are some major open problems in Galois theory?
0
Q: Is it true that the equation $27x^2+1=7^3y^2$ has infinitely many solutions in positive integers $x,y$ ?

Saun DevIs it true that the equation $27x^2+1=7^3y^2$ has infinitely many solutions in positive integers $x,y$ ?

0
Q: What is the value of this improper integral?

Rising Star$\lim_{x\rightarrow 0 } \dfrac{1}{x} \int_{x}^{2x} e^{-t^2} dx$ I dont have any idea to solve this integral ?

0
Q: How to find the height of the frustum? Not the slant height

Ariff ArmanThe two ends of the frastum are circles with radii of 3cm and 9cm. Find the volume of the frastum. How to do the question?

 
5:30 AM
@NormalHuman: Not joining in this time?
 
@MikeMiller Correct. Real life's been busier than last year, so I withdrew from some aspects of SE.
-2
Q: Average Question

SuyogBrian Lara, the famous batsman, scored 6,000 runs in certain number of innings. In the next five innings he was out of form and hence, could make only a total of 90 runs, as a result of which his average fell by 2 runs. How many innings did he play in all, if he gets out in all the innings? (a)...

Welcome to Math.SE, Suyog. Words such as question are uninformative in titles. Please edit the title so that it better describes the specifics of your question. Do not hesitate to make it longer or include a formula if needed. More tips here. (from a bot)Normal Human 22 secs ago
 
0
Q: second quantization and elementray questions

anna vI want your opinions on how useful it is to throw second quantization at students when they have not understood first quantization. I tend to add a simpler answer when I see this. Also the emphasis on second quantization and statements of the type "all space is filled with electron fields /parti...

 
Fair enough. I appreciate the comment bot, by the way.
 
0
Q: Proving $\Box A \rightarrow \Box\Box A$ from $KG_r$

AddemI'm trying to prove $\Box A\rightarrow \Box\Box A$ from $KG_r$ where $KG_r$ is the axiom $$\Box[ \Box A \rightarrow A ] \rightarrow \Box A$$ I'm given the hint that $$ A \rightarrow ((\Box\Box A\land \Box A) \rightarrow (\Box A \land A))$$ is a theorem. I know that in any normal system you...

Questions tend to get more attention when they have a tag for a broad area of mathematics relevant to the question. Some of these tags might fit. (from a bot)Normal Human 20 secs ago
0
Q: Set Theory Proof Verification

Martin Rand Theorem: Let A and B be sets such that A$\cup$B≠B and B\A={}. Then we have that B$\subset$A Theorem is True. A and B are not identical sets. A contains all of B's elements, therefore B is a subset of A. Is this correct?

Short title. Tagged proof-theory. Set Theory Proof Verification
 
5:54 AM
0
Q: I won't be able to earn Wireless or Field Work

Carlos MuñozHaving only Windows Phone as mobile operating system I won't be able to earn any of these two hats: Wireless nor Field Work Is it possible to know when will you guys stop undervaluing the Windows Universal Platform and start working on an app for it? It isn't that hard you know.

 
0
Q: Exponential function (growth question)

user298555The population of an ant colony doubles every day and a half. A biologist predicts the ant colony will eventually overtake a flower bed. What is the number of hours between the time the flower bed is half full and the time the flowerbed is completely full? Explain.

Welcome to Math.SE, user298555. Words such as question are uninformative in titles. Please edit the title so that it better describes the specifics of your question. Do not hesitate to make it longer or include a formula if needed. More tips here. (autocomment)Normal Human 21 secs ago
0
Q: What is the value of the series?

Rising StarWhat is the value of $\dfrac{1}{1!} + \dfrac{1+2}{2!} + \dfrac{1+2+3}{3!} + \dots $ How to proceed with it ?

0
Q: An MCQ on Greens function

A RG(x,t) =\begin{cases} a+ b(logt) & \text{if $0<x<t$ } \\[2ex] c+ d(logt) & \text{if $t<x<1$ } \end{cases} is a Greens function for $xy''+y'=0$ subject to y being bounded as x tends to $0$ and $y(1)=y'(1)$ if a) a=b=c=d=1 b)a=c=1,b=d=0 c)a=c=0,b=d=1 d)a=b=c=d=0 I don't know how to tackle th...

 
6:18 AM
0
Q: How many solutions does the equation $\sum_{i=1}^{k}{x_i}=c$ have, given that $x_i$ are integers and $0\leq x_i\leq d$?

user1145925We are given initially some $k,c,d\in\mathbb{N}$. How many solutions $(x_1, x_2, ..., x_k)$ does the equation $\sum_{i=1}^{k}{x_i}=c$ have, where $x_i\in\mathbb{Z}$ and $0\leq x_i\leq d$?

 
6:29 AM
0
Q: L'Hopital's Rule

Paula B.I am a Calc I student having trouble understanding why exactly L'Hopital's rule works and when exactly it can be used. I know this question is general, I just want to gain more of an understanding of the rule...thank you for your time.

Short title. L'Hopital's Rule
 
6:40 AM
0
Q: Integral of a piecewice trigonometric function

BahaThere is an expression given for a piece-wise linear function, fn(Phi). fn is being a function of Phi: u = u0*cos(w*t-y) = u0*cos(Phi) where Phi = w*t-y Assume u0, w, t are positive Assume u0 > Fy/kf fn(Phi) = Fy + kf * (u - u0) for 0 <= Phi < Theta fn(Phi) = -Fy for Theta <= Phi < pi fn(Phi...

0
Q: How to find an integer $x$ such that $140x \equiv 133 \pmod{301}$

E. A.How to find an integer $x$ such that $140x \equiv 133 \pmod{301}$? There is a hint that $gcd(140,301)=7$

0
Q: Question over Appolonius CCC problem (analytic geometry)

WillemienI have 3 circles: $C_1$ centered at $(0,0)$ with radius 1 $C_2$ centered at $(a,0)$ with radius $a+1$ $C_3$ centered at $(-a,0)$ with radius $a+1$ (so $C_1$ is internaly tangent to both $C_2$ and $C_3$ ) Question: What are the centres of circle $C_4$ and $C_5$ that are tangent to all three ...

Words such as question do not add information to titles. Please edit the title so that it better describes the specifics of your question. Do not hesitate to make it longer or include a formula if needed. More tips here. (autocomment)Normal Human 22 secs ago
0
Q: Value of $\lim_{n\to \infty}(\frac{1}{n}+\frac{1}{n+1}+\frac{1}{n+2}...+\frac{1}{2n})$

AmartyaWhat is the value of $\lim_{n\to \infty}(\frac{1}{n}+\frac{1}{n+1}+\frac{1}{n+2}...+\frac{1}{2n})$? Please give some hints to proceed .

0
Q: I wan't to check for the convergence/divergence of $cos(1/n)$?

Borisfor checking it $$lim_{n \to \infty}cos(1/n)=lim_{n \to \infty} \frac{e^{i/n}+e^{-i/n}}{2}=(1+1)/2=1$$ this means that the series diverges is it right thought.

 
1
Q: Where to ask mobile communication questions?

dheeraj92Actually I want to ask questions about interfaces required for communication between network entities in mobile communication.

 
0
Q: problems solvable in polynomial time

cgoWhat is the origin and context of the phrase 'solvable in polynomial time' in computer science? Are they related to the notion of 'polynomials' in mathematics?

Title contains problem. Short question. problems solvable in polynomial time
0
Q: Solve a system of ODE's using backward euler's method

ChrisKI am trying to solve a system of ODE's using the backward euler's method in MATLAB. The system has 3 equations with the initial conditions given below. x' = λ - ρx - βxz; y' = βxz - δy; z' = py - cz; x0=43100; y0 = 0, z0 = 0.0033, λ = 388, ρ = 0.009 δ = 0.18, p = 50000, c = 23, β=3.61e-8 Can s...

Question contains please. Tagged matlab. Solve a system of ODE's using backward euler's method
0
Q: Is $A$ a finite set?

AmartyaLet $A=\{\sum _{i=1}^\infty \dfrac{a_i}{5^i}:a_i=0,1,2,3,4\}\subseteq\Bbb R$. Then is $A$ a finite set? the series $\sum _{i=1}^\infty \dfrac{a_i}{5^i}$ always converges to $0$ .So $A=\{0\}$ Is it correct?

Short title. Is $A$ a finite set?
 
7:24 AM
0
Q: Is there any contradictory that both flagging NAA and leaving comment for newbies?

GstestsoSometimes there may some newbies that post comments as answers. If I flag them as NAA, they will be cleared but the user may never know what they did wrong. On the other hand, if I just leave comments but not flagging them, they can see comments and know what they did wrong. My question is, is t...

 
0
Q: Uniformly distributed

Brian ByrneSuppose that X is uniformly distributed on (0,2). A) Find E[X] I got that to equal to 1 B) Find E[e^X] I got that to equal to 3.1946 I am wondering if these are right and if not could you please show me how to do it the correct way.

Short title. Question contains please. Uniformly distributed
0
Q: What is E[X] of V^2 given that V is a normal random variable with mean = 0 and standard deviation = sigma

DrewI am trying to compute the expected value of a function V^2, given that V is a normal random variable with mean = 0 and variance = sigma squared?

Short question. [What is E[X] of V^2 given that V is a normal random variable with mean = 0 and standard deviation = sigma](math.stackexchange.com/q/1574848)
 
-1
Q: How can I use Stack Exchange Q&A as a suggestions in my website

RajI wanted to use Stack exchange network as a suggestion like here in google https://cloud.google.com/appengine/docs/java/xmpp/?csw=1 How can I achieve this ? Is that any open source API from which I can enable this in my Website? Any help is most welcome.Thanks in advance.

 
7:44 AM
0
Q: Intersection of dense subspaces of a Hilbert space

Kavi Rama MurthyDo there exist two dense sub-spaces of a Hilbert space whose intersection is {0}?

0
Q: About a "99 similar and 1 not similar" problem

Shivam PatelA great friend of mine recently sat for an interview. He was asked a question which has fascinated me since some days now. It is Consider you have 100 balls which look the same but one out of them is either heavier or lighter than the rest . You are given a beam balance , then find out in mi...

0
Q: V is modeled as a normal random variable with mean 0 and variance σ 2. What is E[X^2]

user298565I am trying to find the expected value of the function Z = mV^2/2, in terms of sigma and m, where m is a constant greater than 0.

Short question. [V is modeled as a normal random variable with mean 0 and variance σ 2. What is E[X^2]](math.stackexchange.com/q/1574870)
 
0
Q: Should I reduplicate or edit a poorly worded question with no good answers?

neuronetI have a question that has been asked twice already (namely, a feature request to be able to link to specific points within a question or answer). The problem is the second was marked as a dupe of the first (even though the first has no good or accepted answer and was fairly poorly worded). It al...

 
8:02 AM
0
Q: Why the fucntion $\sin(1/x)$ is not bounded around zero?

AnixxIntuitively it seems that function $\sin(1/x)$ should be bounded, but analysis shows it is not the case. The function takes arbitrarily large values around zero. Why it is the case?

Consider replacing (analysis) with a more specific tag for the relevant branch of analysis. (autocomment)Normal Human 50 secs ago
0
Q: Presentation of Groups

PeterI have troubles to solve this kind of exercises. For example, let $G_1=<x,y |x^3=y^4=1> $ and $G_2=<x,y |x^6=y^6=(xy)^3=1> $. I want to check that 1) $G_1$ is infinite and nonabelian and 2)$xy^2x \neq 1$ in $G_2$. For the first part, I have seen that it is useful to define a group homomorphism...

0
Q: No. of distinct positive integral solutions

MD SHAHBAZ SHAFIWhat is the total no. of positive integral solutions of: X1 + X2 + ... + Xk-1 < Xkwhere Xi < Xi+1 and each of Xi <= N

Welcome to Math.SE, MD SHAHBAZ SHAFI. This site uses MathJax formatting of formulas. More tips here. (from a bot)Normal Human 1 min ago
0
Q: A system of equations

Ali BagheriLet $H$ be a non-separable Hilbert space. Assume $E$ is an orthonormal basis in $H$. Let $E_0=\{e_n\}$ be a countable subset of $E$ and let $\{\zeta_n\}$ be a bounded sequence in $H$. Let $E_1$ be a countable susbet of $E$ and denote $p$ by the projection onto $\overline{span\{e: e\in E_1\}}$. ...

0
Q: Urn Probability Problem (conditional relacement)

Jill ZegaI am working through Parzen and I came across a problem that has completely stumped me. I have an urn which has M black balls and N white balls. Each turn, I randomly reach in and choose one ball without replacement. If the ball is black, I add on white ball to the urn (and no not replace the dra...

0
Q: Optimization problem for a piecewise constant function

PAMI am considering the following optimization problem: $\underset{u}{\text{minimize }}\quad \underset{v}{\max} f(u,v)$ where for a fixed $u$, the function $f(u, .)$ is piecewise constant. Do you know if there exists any simple approach to solve this kind of problem? Which one should I use? Than...

0
Q: Integrating $e^x^x$ .

user10859$\int e^{x^x} $. taking u = $x^x$ so du= $x^x$[logx +1] dx -> $\frac {du}{[u(log(x) +1)]}$=dx Now to remove log(x) from denominator I took log(u)= x log(x) -> log(x) = $\frac {log(u)}{x}$ Again If I use that log(u) to remove x I get log(x). How to proceed from here, Or is there another me...

 
8:47 AM
0
Q: How do I migrate a post with open bounty from SO to Android Enthusiasts?

naXaI have gone through similar questions on meta but as it's still not clear I am asking a different one. Here is a post which is completely off-topic. It's asking about finding an existing Android app and doesn't suit the Stack Overflow (i.e. site for those who code). It is best suited for Android...

 
0
Q: Computation of residue class of $2^{100}$ modulo $1000$

Randy Randerson$2^{100} \equiv 1 \ (\text{mod}\ 125)$ and is divisible by $8$. Why then is $2^{100} \equiv 376 \ (\text{mod}\ 1000)$?

 
8:59 AM
1
Q: Track winter bash hat

Anik Islam AbhiLike previous year , Winter bash also comes this year with hats. Every hat has a challenge to achieve it.It's really fun to collect hats . But there is no tracking system like tag,badges to track next hat. As Winter bash comes every year, can it be possible to open a tacking system of hats lik...

0
Q: What is this "Winter bash" and why did I get a cake on my profile picture and a hat in my profile?

mark kirbyTitle says it all really, logged in today, got a hat on my profile activity page, a cake to customize my profile picture and a new snowflake icon on the notification bar. So what is winter bash?

 
0
Q: How does one read $\bar{A}$ aloud in Russian?

DavidIn English, I would generally read the mathematical notation $\bar{A}$ as "a bar" or "big a bar." What would this be in Russian? Thanks.

Questions tend to get more attention when they have a tag for a broad area of mathematics relevant to the question. Some of these tags might fit. (from a bot)Normal Human 1 min ago
0
Q: Conditional expectation $E[\min{\{X,Y\}}|Y^{2}]$

LeonLet $X$ and $Y$ be an independent random variables with exponential distribution $Exp(1)$. Compute $E[\min{\{X,Y\}}|Y^{2}]$. My attempt: I. If we want to compute $E[\min{\{X,Y\}}|Y]$ may be sufficient to use the following formula: $$E[f(X,Y)|Y]=\int\limits_{-\infty}^{+\infty}f(x,Y)f_{X}(x)dx$...

Short title. [Conditional expectation $E[\min{\{X,Y\}}|Y^{2}]$](math.stackexchange.com/q/1574928)
0
Q: Degree of splitting field for $x^3-3x-1$ over $\Bbb Q$ and $\Bbb F_5$

Makoto K.I want to find the degree of the splitting field for $x^3-3x-1$ over $\Bbb Q$ and $\Bbb F_5$. My attempt is contained below.

0
Q: Bernoulli random variables

Brian ByrneLet X1,X2,X3,... be an i.i.d. sequence of Bernoulli random variables where Xi=0 with probability 0.5 and Xi=1 with probability 0.5. For a fixed value of N, define Xbar = X1+X2+X3+...+XN/N A) Find u(subscript Xbar) = E[Xbar] and Sigma^2(subscript Xbar)= Var[Xbar]

This site uses MathJax formatting of formulas. More tips here. (autocomment)Normal Human 47 secs ago
0
Q: V is a normal rv with mean 0 and variance 2. Find the PDF of the function Z = mV^2/2

user298577How can I find the PDF of the function Z=mV^2/2, in terms of sigma and m, where m is a constant greater than 0?

 
9:33 AM
0
Q: Bilinear form over C

byteSlayerLet $f:V\times V \rightarrow C$ be a bilinear form in a finite inner product space. Will there always be a single linear transformation $T:V\rightarrow V$ for which $f(v,u) = <Tv,u>$ for each $v,u\in V$. If not - what's an example of this not happening? (I can prove this for bilinear forms $f:V\...

Short title. Bilinear form over C
 
0
Q: Bug in showing disputed flags

NSNoobI saw this question in triage today. After reading initial lines, It appeared to be spam (Discussing his website with details of its services in a promotive narrative). So I flagged it as spam. After a while, I realized my mistake and flagged it as off-topic because the guy was asking for recomme...

-1
Q: Why is my question on hold?

DavidI asked this question: How does one read $\bar{A}$ aloud in Russian? How does this decision fit with the majority view here: Questions about how to read mathematical notation If possible, I would like to have the question reopened. Thank you. Edit: I've received a good answer in the comments t...

 
0
Q: How does the internal language of a topos come to be?

ArrowThere are several books and articles on topos theory which mention the internal language, but I can't manage to see the big picture from any of them. I would like a soft explanation of how the entities in the definition of an elementary topos come to together and give rise to this internal logic....

Consider adding a tag for a broader subject area to which the question belongs. Some of these tags might fit. (autocomment)Normal Human 20 secs ago
0
Q: Solve the integral $\int x^2 e^{-\frac{x^2}{2}}dx$

Math 1988Please solve the following integral.$$\int x^2 e^{-\frac{x^2}{2}}dx.$$

Short question. Question contains please. Solve the integral $\int x^2 e^{-\frac{x^2}{2}}dx$
0
Q: Why does $3f_3 + 4f_4 + 5f_5 + ...= 2E$

Sean MorrisWhy does $3f_3 + 4f_4 + 5f_5 + ...= 2E$? Is there a specific proof for this or is it just due to the fact that each edge makes up parts of 2 different faces. I have looked all over the internet and I cannot find a proof online, everyone just seems to 'assume' that it does. Please help. Thanks

Question contains please. Why does $3f_3 + 4f_4 + 5f_5 + ...= 2E$
 
9:53 AM
-2
Q: UCS SDK_v4.3.4.0 Server Failed to connect

Samir AlghazalyI have a problem with unis 3.1.1 on Virdi AC7000 when transferring users data from terminal to another all user data is transferred very well but without finger print data so I tried to use the UCS SDK_v4.3.4.0 (UCBioBSPCOMLib,UCSAPICOMLib) and when trying to start my app as a server it gives me ...

0
Q: Is this question migratable to MO?

Rajesh DachirajuIs it worth migrating this question to MO, from math.SE? It seems elementary but did not get any response on MSE even after 10 days.

 
10:03 AM
0
Q: A 'weird' topology: Hausdorff, quotients.

JBIBBLet $X$ be obtained by taking two disjoint copies of the interval $[0,2]$ (with the Euclidean topology) and gluing each $t$ in the first copy with the corresponding $t$ in the second copy, for all $t \in [0,2]$ different from the middle point. Explicitely, one may take the space $$ Y=[0,2]\times...

0
Q: CW complexes vs simplicial

in and out o' mindHow are simplicial complexes more helpful than CW complexes with respect to smooth manifolds? What are some of the benefits of one vs. the other?

Short title. Short question. CW complexes vs simplicial
0
Q: jhghjgvgjffgghfghf

User9523Considering statements (1) , (2) , (3) & (4) to be true , $$ \sum_{j=1}^B (w_kj + y_kj) + (e_k - b_k)a \leq f_l$$ $$ \sum_{j=1}^B (w_kj + y_kj) \leq f_l - (e_k - b_k)a $$ $$ \sum_{j=1}^B (w_kj + y_kj) \leq f_l + (b_k - e_k)a $$ Summing up in terms of $k$ , $$ \sum_{k=1}^{4}\sum_{j=1}^2 (w_...

Short title. jhghjgvgjffgghfghf
 
10:28 AM
0
Q: Doing it myself for the hat!

KarmaEDVJust do it. Swoosh. http://winterbash2015.stackexchange.com/

 
0
Q: Relation between two numbers

HemantIf $x=\pi^{3}$ and $y=3^{\pi}$ then what is the relation between x and y? I have no idea of how to solve this problem. Please help.

Short title. Short question. Question contains please. Relation between two numbers
 
10:47 AM
0
Q: Where am I wrong in this "proof" that the collection of sets is countable?

drhabLooking at sets (in ZF, Set Theory of Kunen) I could not escape from looking at logic as well. A formal language is presented containing basic symbols ($\wedge,\neg,\exists,(,),\in,=,$ and $v_{i}$ for $i=0,1,\dots$). Also things are said about free variables and bound variables. Formulas are d...

0
Q: Ito integral via simple process when the integrand is C^1

Fred G.I have the following problem. Let $H_t$ be an adapted process with trajectories a.s. of class $C^1$ on $\mathbb{R}_{+}$. Compute using simple process $\int_o^t H_s d B_s$. My idea is to firstly set $U_n(s):=\sum_{i=0}^{m(n)-1} H_{t_i^n} \mathbb{1}_{(t_i^n-t_{i+1}^n]}$ (where of course I consider...

 
0
Q: Useless edits in the review queue

mark kirbyI know this comes up all the time but I just want to bring it back one more time. When low rep users edit questions and answers can they please make sure they make they actually contribute to the post with there edits. Today, again, the edit queue is full of one tag edits and, most annoying of ...

0
Q: How to deal with outdated answer for old question

VoidPointerCustom HTTP method with Nodejs HTTP Server Answer to this question is modifying piece of code in older version of NodeJS, which was working for OP. But with latest version NodeJS files, the accepted answer is no longer relevant. How can I get answers for latest versions? Post new/similar questi...

-2
Q: What should I do for my these questions?

Mithlesh UpadhyayI've done some mistakes on Stack Overflow regarding to post some questions, which may not not be there. They not recieved well questions, so they gave me votedown, e.g. I'm missing something Swapping strings in C [duplicate] How we 5 replaced by 27, since there is no relationship? Is it possib...

0
Q: Winterbash Hats: localized description

AShI have visited http://winterbash2015.stackexchange.com/ to see more about hats It shows me localized description for hats (Russian). A bit strange after English-only SO.com, I'm Ok with English, but thanks anyway But some translations hurt because of typos I did search but couldn't find where ...

 
0
Q: What is an outer measure?

Al jabraI know the definition of an outer measure. Let X be a set. An outer measure on X is a function $\theta : \mathcal{P}X \to [0,\infty]$ such that (i) $\theta \phi=0$ (ii) if $ A \subseteq B \subseteq X$ then $\theta A \leq \theta B$ (iii) for every sequence $<A_n>_{n \in \mathbb{N}}$ of subsets...

 
11:12 AM
0
Q: Serial close votes on pronunciation questions

DavidI have noticed that since, an hour ago, I asked for my question on how to read a formula aloud to be reopened ( Why is my question on hold? ) some users have begun casting close votes on dozens of old questions on pronunciation. As far as I can tell, there has been no open discussion of this on ...

 
11:28 AM
0
Q: Proof: Angle between two diagonals of a parallelogram

BahaCan anyone prove the angle theta (smaller angle) between diagonals of a parallelogram is given by the equation cos(\theta)=\frac{a-c}{a+c} where "2*a" is the base length of parallelogram; "2*c" is the horizontal distance to the edge when we draw a vertical line as seen in the picture below. ...

0
Q: Existence of derivative of a given function

MarcGiven $$F(x)=\frac{1}{2x}\int_{-x}^xf(t)dt,$$ where $f:\mathbb{R}\to\mathbb{R}$ is continuous and $f'(0)$ exists, how can I prove that $F'(0)$ also exists?

0
Q: How does $< v , e_j> = 0 \implies v =0$ implies $v \in span(e_j)$?

HawkLet $H$ be a Hilbert space. Some books define an orthonormal basis as one that is in the title. I am using the standard linear algebra definition, that is, an orthonormal basis is one that is a basis and orthonomal in the linear algebra (finite-dim) sense (every element in $H$ is a linear (poss...

 
11:52 AM
0
Q: Let $A=\{\sum_{i=1}^{\infty} \frac{a_i}{5^{i}}:a_i=0,1,2,3$ or $4 \} \subset \mathbb{R}$. Then which of the following are true??

tattwamasi amrutamLet $A=\{\sum_{i=1}^{\infty} \frac{a_i}{5^{i}}:a_i=0,1,2,3$ or $4 \} \subset \mathbb{R}$. Then which of the following are true: a. $A$ is a finite set. b. $A$ is countably infinite. c. $A$ is uncountable but does not contain an open interval. d. $A$ contains an open interval. Each such seri...

 
1
Q: Why is seeing updates of new answers to old questions an exclusive privilege?

BazzI came across this question on meta recently: Make the New Answers to Old Questions real time It got me thinking: why can't newer users see new answers to old questions alerts (10000 rep required)? As Stack Overflow becomes full of good general questions, I think the scope for good non-duplicat...

0
Q: Custom tab doesn't show questions

stefan.s 웃I created a custom tab, consisting of the following tags: windows-phone-8 windows-10 windows-10-mobile windows-universal win-universal-app all combined with any. However if I open up the tab, I just see the message "That's a miss" Strangely if i set the cursor in the edit field and move it to...

 
0
Q: Existence of the Schiffler Point

user298600Let triangle $\triangle{ABC}$ have incenter $I$. Prove that the Euler lines of $\triangle{AIB}, \triangle{BIC}, \triangle{CIA}, \triangle{ABC}$ are concurrent.

0
Q: Caratheodory's method

Al jabraThe theorem is: Let X be a set and $\theta$ an outer measure on X. Set $\sum= \{E: E \subseteq X, \theta A=\theta(A \cap E)+ \theta(A \backslash\ E)$ for every $A \subseteq X\}$ What is the intuition behind this definition? and what does the Caratheodory method try to achieve?

 
12:22 PM
0
Q: Is the root of all numbers who have a lone prime factor irrational?

Nimitz14I've got what I think is a proof, am wondering if I've made a mistake. Proof by contradiction: Suppose sqrt(n)=a/b, with a and b being integers and coprime meaning a/b is rational. Square it, so n=a^2/b^2 and take the b^2 over: b^2 n = a^2 The way I see it, since a^2 is taking the prime factor...

This site uses MathJax formatting of formulas. More tips here. (autocomment)Normal Human 21 secs ago
 
0
Q: "x votes left today" dialog close button is positioned wrongly

ArtOfCodeWhen you have <= 5 votes left in a day, there's a mini-dialog that pops up informing you of this fact. There's a button on the end of it that lets you close it early, before it automatically closes. This button is usually centered vertically in the box, but today it's broken: The same is appar...

 
0
Q: What is a constant field?

Mary StarI am looking at the following: Could you explain to me what a constant field is?

Short title. Short question. What is a constant field?
 
0
Q: Is the expandable usercard available to everyone?

TheLethalCoderAccording to the help page on privileges. In particular Established user. The user receives the ability to see expandable user cards on other users. This appears when the user reaches 1000 reputation. However I am only at just over 700 and I am able to see them. Is this a bug or something that w...

 
0
Q: How to integrate with reduction formula?

Nataliaenter image description here How to integrate (cos x)^n.(sin x)^m with reduction formula. the actual steps please in terms of (n-2,m) and in terms of (n, m-2) like in the image shown I'm stuck halfway Thanks https://en.wikipedia.org/wiki/Integration_by_reduction_formulae

0
Q: Find the value of the series $\sum_{i=1}^ \infty \dfrac{n}{2^n}$

AmartyaFind the value of the series $\sum_{i=1}^ \infty \dfrac{n}{2^n}$ The series on expanding is coming as $\dfrac{1}{2}+\dfrac{2}{2^2}+..$ I tried using the form of $(1+x)^n=1+nx+\dfrac{n(n-1)}{2}x^2+..$ and then differentiating it but still it is not coming .What shall I do with this?

Tall formulas in titles break the layout of question lists. Please replace \dfrac with \frac in the title. (autocomment)Normal Human 21 secs ago
0
Q: Intersection theory question from Vaki'ls notes on Algebraic Geometry

user45150I cannot seem to solve what I think should be a straightforward intersection theory question in Ravi Vakil's algebraic geometry notes. Let $X$ be a scheme and $Y$ a closed subscheme of dimension less than or equal to $n$, and $\mathscr{L}_1,\ldots,\mathscr{L}_n$ be a collection of line bundles o...

0
Q: AB is parallel to CD. What is the value of y?

DylI really need help on this question. diagram: http://media.apexlearning.com/Images/200702/13/1ef91461-c2f5-4d1a-9188-5c5fe1fd849a.gif

1
Q: How would I approach this optimization question?

Mone Skratt HenryThe question reads : A box (with no top) is to be constructed from a piece of cardboard of sides A and B by cutting out squares of length h from the corners and folding up the sides as in the figure below: Suppose that the box height is h = 3 in. and that it is constructed using 134 in.^2 of c...

Words such as question do not add information to titles. Please edit the title so that it better describes the specifics of your question. Do not hesitate to make it longer or include a formula if needed. More tips here. (from a bot)Normal Human 20 secs ago
0
Q: Need help understanding the solution of: Show that there is no perfect square whose last three digits are 341.

Chaow WuShow that there is no perfect square whose last three digits are 341. Solution: Let x be any integer. Note that if $x^2$ ended with the digits 341, then we would have $$x^2 = 1000k+341$$ for some integer k, so $$x^2 = 1000k + 341 = 341 = 5 (mod 8)$$. We make a table of squares modulo 8. $$ x: ...

0
Q: Proving First Order Differential Equations

Simon KupaliaAn equation of motion when a particle moves in a resting medium is given by dv/dt=-(kV+bt) where k and b are constants.Given that v=u,when t=0..show that v=b/k^2-bt/k+(u-b/k^2)e^-kt

Welcome to Math.SE, Simon Kupalia. This site uses MathJax formatting of formulas. More tips here. (from a bot)Normal Human 21 secs ago
 
1:05 PM
0
Q: Challenge and Interesting point - GLM for Poisson Regression for Titanic survival data

Jay tesI have a professor who made a very good point about the data titanic analysis during a lecture this week. I am still however trying to better understand. He argued that it is also possible to have a model of titanic data (GLM) using Poisson rather than the typical Binomial approach. I am curious ...

0
Q: Ordered Pairs and Set Theory

Damjan BabićSo, first of all I'm gonna start with stating that I'm a complete noob in mathematics that started doing this as a hobby. I encounter problems that I can't figure out on a regular basis, like a few times per page but I usually find someone to help me. This time those same people were of no help s...

0
Q: Prove that if $n$ is a positive integer greater than one which is not prime then it is divisible by some prime $p <= sqrt(p)$

GeoffI have just started revising number theory and I am getting stuck on a lot of the "prove that" questions. Any tips and advice would be much appreciated!

0
Q: What is the difference between antiderivative and derivative?

MelissaI am in calculus class right now and I have no idea. I'm sorry for my ignorance.

 
0
Q: Неправильный перевод на кнопке в новых Шляпах

xajaПо правилам русского языка — «надеть шляпу»

 
1:24 PM
0
Q: Isometries of reflections

mathnlatteI'm totally stuck at this problem and any help would be appreciated: Suppose $R_\rho \in M_2$ denotes a reflection across a line which is through the origin and at a anticlockwise angle of $\rho$ with the x-axis. The question asks that for any $n \in \mathbb{N}$, does there exist a subgroup of $M...

0
Q: How Many Primes Are There?

JqueryKingOver 2,300 years ago Euclid proved that the number of primes is infinite, so two possible questions come to mind: How many primes are there less than the number x? There are infinitely many primes, but how big of an infinity? This document will focus on the first question. The seco...

0
Q: Rate of Change Questions?

Marygrace If h(t) represents the height of an object above ground level at time t and h(t) is given by h(t)=-16t^2+13t+1 find the height of the object at the time when the speed is zero. Suppose h(t)=t^2+14t+7 . Find the instantaneous rate of change of h(t) with respect to t at t=2 . Suppose G(x)=6x^2+x+...

This site uses MathJax formatting of formulas. More tips here. (autocomment)Normal Human 21 secs ago
0
Q: Find inverse function of N*N

user290335Hi can anyone help me on this problem? In have the function 2^n*(2n+1)-1 and I need to prove that the function has an inverse function. So I did: f(x,y)= 2^n*(2n+1)-1 f(x,y)=t, where t=59 so: 2^n*(2n+1)-1 =59 2^n*(2n+1)=60 (Factorize 60 it becomes) 2^n*(2n+1)=2*2*3*5 after that I am los...

This site uses MathJax formatting of formulas. More tips here. (autocomment)Normal Human 21 secs ago
0
Q: Variation of Constant for

the.polo$$x''=-2\beta x'-x+\gamma\cos(\omega t)$$ with $\beta,\gamma,\omega$ positive constants. I started by finding the homogeneous part of the solution by solving $x''+2\beta x'+x=0$ and found $$x_{hom}=c_1e^{\lambda_+ t}+c_2e^{\lambda_- t}$$ with $\lambda_{\pm}=-\beta\pm\sqrt{\beta^2-1}$. Now to find...

0
Q: I am stuck on a semi-infinite strip, and am trying to get to the upper half plane,

User001The problem statement is: Let D be the domain consisting of all points satisfying Im $z>0$ and (Re $z)^2$+(Im $z−1)^2>1$. Map $D$ onto the upper half plane. So the domain is everything on in the upper half plane, minus the circle of radius 1, centered at $(0,i)$. I first applied the logarit...

0
Q: Closure in profinite topology

NilsLet $G$ be an abstract group (say, finitely generated). Endow $G$ with the profinite topology. I would like to get comfortable with computing the closure of subgroups of $G$. More specifically, I would like to understand how to use the profinite completion of $G$ to do that. If $G$ is residual...

Welcome to Math.SE, Nils. Consider adding a tag for a broader subject area to which the question belongs. Some of these tags might fit. (from a bot)Normal Human 21 secs ago
Title contains problem. Tagged proof-verification. Compound Interest : Clarification Of The Given Solution To This Problem.
0
Q: Positivity of this improper integral

userI am not relieved of the following problem and trip. Let $\varepsilon>0$ be a small parameter, $a>0$ be a given constant, $x_{\varepsilon}\in(0,a)$ be a given sequence such that $x_{\varepsilon}\to a$ as $\varepsilon\to0$ and $f:[0,a)\times[0,a)\to\mathbb{R}$ be a continuous function such that $...

 
1:43 PM
1
Q: Allow rooms to opt-out of gist previews

Hunter StevensWe've noticed this new feature in the Ruby chat room this morning: http://chat.stackoverflow.com/transcript/message/27523117#27523117 If you check our room's transcript, you'll see that a couple minutes before I deleted a message, which contained the same code from the gist. When sample code ...

 
0
Q: Probability to pass an exam, generalized competition problem.

curiousIn an exam there are 10 questions. If you answer correctly to a question, you get $1$ point. If you answer incorrectly to a question, you get $-1$ point, or lose a point. If you don't answer to a question, you get $0$ point. You pass the exam if you get at least $7$ points. A pupil read the ques...

Welcome to Math.SE, curious. Tag (contest-math) should not be the only tag a question has. Please add a tag for a subject area to which the question belongs. (autocomment)Normal Human 21 secs ago
0
Q: A problem concerning catalan number

Vamsi SpidyShow that the number of ways to stack coins with n coins in the bottom row is denoted by Catalan number Dn = (2nCn)/n+1. I have tried in the way described in the picture below

Title contains problem. Short question. A problem concerning catalan number
0
Q: Parallel Vectors

bulbasaur Found this on a textbook and I know I am missing out something really simple as it's been a while for me with vectors so here it goes:If $ \Bbb P=mi-2j+2k$ (where i,j,k are unit vectors) and $ \Bbb q=2i-nj+k $ are parallel to each other, then find $m$ and $n$

Short title. Parallel Vectors
0
Q: For how high a degree is the following formula exact?

mazieintegral from -1 to 1 f(x) dx = (5/9)f(-sqrt(3/5))+(8/9)f(0)+(5/9)f(sqrt(3/5)) verify your answer by continuing the method of undetermined coefficients until an equation is not satisfied. Now im stuck, i dont really understand Gaussian quadrature and can some one help me use the method of undet...

This site uses MathJax formatting of formulas. More tips here. (from a bot)Normal Human 21 secs ago
0
Q: Show $E((X-Y)Y)=0$

ketum If $EX^2<\infty$ and $E(X|\mathcal G)$ is $\mathcal F$-measurable then $E(X|\mathcal G)=EX$ There is one step in the proof which I don't understand, set $Y=E(X|\mathcal G)$ and then why is $E((X-Y)Y)=0$, from a theorem I know that $Y\in\mathcal F$ is such that $E(X-Y)^2$ is minimal

Short title. Show $E((X-Y)Y)=0$
 
2:13 PM
-2
Q: How to read file and load an object / the file has an odd format

HuntI have been given a file to read and load the properties of an object the file seems not to be a standard format it just has text sections indicate the properties; I was expecting a sort of xml file format. Does anyone know this type of file and how to read it? each property has a "-- textlab...

 
0
Q: Why $[0,1]$ is not conex

mattI try to prove that $[0,1]$ is conex.I have to follow the next steps: $R$ is complete using the construction of $R$ with Cauchy sequences $\Rightarrow$ $[0,1]$ is complete ,$[0,1]$ compact$\Rightarrow$ $[0,1]$ is conex.How can I prove that?

Short title. [Why $[0,1]$ is not conex](math.stackexchange.com/q/1575208)
0
Q: Wave equation with angular variable

BowParkSuppose that the variable of a mono-dimensional wave equation is an angle: $$\frac{\partial^2 f(\phi)}{\partial \phi^2} + k_{\phi}^2 f(\phi) = 0$$ This equation is derived from a more complex (and separable) equation in cylindrical coordinates and $\phi$ is one of these coordinates. According t...

Tagged differential-equations but mentions "partial". Wave equation with angular variable
0
Q: Galois group of $x^6-2$ over $\Bbb Q$

Makoto K.I want to find the Galois group of $x^6-2$ over $\Bbb Q$. I have posted my attempt in an answer below. Is there a better way? Alternative proofs are greatly appreciated.

0
Q: Poisson Distribution# Statistics

PrecisionPoisson distribution with a mean of 2 books bought per hour. (a) What is a probability that no one will buy a book in a given day (24 hours)? (b) The probability that there is at least one book bought in the next minute?

0
Q: What does this limitation mean?

userIn a book I'm reading, the following statement is written: Let $\varepsilon>0$ and $\delta>0$ be parameters with $\varepsilon<\delta$, $\{x_{\varepsilon}\}_{\varepsilon>0}\subset(0,1)$ be a sequence such that $x_{\varepsilon}\to a\in(0,1)$ and $f:(0,1)\to\mathbb{R}$ be a continuous function. ...

Consider replacing (analysis) with a more specific tag for the relevant branch of analysis. (from a bot)Normal Human 21 secs ago
 
3
Q: Increase the gap between "Award a bounty" buttons

alecxeIf an answer was previously awarded a bounty, the next "Award a bounty" button is displayed below without any gap: (link to the thread). Not sure if I should've tagged this as a bug.

 
0
Q: Angle of the diagonal of a convex quadrilateral.

Manolis KalafatisGiven a convex quadrilateral in which we know the angles of the corners and of the one diagonal, can we find the angle of the second diagonal? quadrilateral

0
Q: Let $f: \Bbb R \to \Bbb R$ be a continuously differentiable function such that $|f(x)-f(y)| \ge |x-y| \forall x,y \in \Bbb R$ Then $f'(x)=\frac 12$

Vikrant Desai A. has exactly one solution B. has no solution C. has a countably infinite number of solutions D. has uncountably many solutions I choose option C as the answer. This is because $\frac {|f(x)-f(y)|}{|x-y|} \ge 1 \forall x,y \in \Bbb R \Rightarrow |f'(x)| \ge 1 \forall x \in \...

0
Q: Theorem 3.25 from baby Rudin

Raheem Najib It's theorem from PMA Rudin. But I think that we can conclude more stronger conlusion: that $\sum |a_n|$ converges i.e. $\sum a_n$ converges absolutely. Am i Right?

Short title. Short question. Theorem 3.25 from baby Rudin
0
Q: unbiased estimator

user298629Let $\mu,\alpha_n:\mathbb R^+\to \mathbb R$ continuous function with $\mu$ bounded function. Let $N^{(n)}$ the trajectory of a Poisson process with intensity $(\alpha_n \mu)(t)$. Let $0=T_0^{(n)}<T_1^{(n)}<..$ jumps of $N^{(n)}$ Let $M_n(t)=\sum_{i=1}^{N_t^{(n)}} \frac {1} {\alpha_n (T_i^{(n)})}...

Short title. unbiased estimator
 
2:36 PM
0
A: Просьбы посоветовать книгу и т.п

QwertiyЕсли есть подходящий канонический ответ, то закрывать как дубликат. Если нет, то оставлять. Хоть какой-то контент наберётся, а потом можно будет и эталонный ответ собрать. А этот закрыть. Что касается ссылок, то для книг названия достаточно, чтобы найти, поэтому ссылка является просто дополнител...

0
Q: Ping the OP from an answers comments

TheLethalCoderAs far as I can tell this is not a feature. But I can see it being useful to be able to ping the questioner from one of the answers comments. Obvisouly this is only needed if the OP hasn't already interacted with this answer in the normal ways. For example if people are having a discussion it mig...

 
0
Q: Use Nested Interval Theorem to prove that f is bounded on a closed interval $[a,b]$

user289293 Attached is my proof. Is there any flaw in my proof? Thanks in advance.

Short question. [Use Nested Interval Theorem to prove that f is bounded on a closed interval $[a,b]$](math.stackexchange.com/q/1575247)
0
Q: Determing Area Bounded By a Curve Using Double Integration

Simon KupaliaUse double integral to determine the area bounded by the curve y=x^2 and line y=2x+3

 
1
Q: Answering a question after it has been flagged as a potential duplicate. Bad form?

sstanContext So, I came across this question: For Each control not returning all controls. After a quick search, I realized that the question was essentially a duplicate of this question: VB.NET Loop through controls in a panel skips controls, which had answers that would solve OP's question. So I ...

 
0
Q: How can I draw $L_2$ norm, $L_1 $ norm and$L_{\infty }$ norm by using matlab.

MOPFor the closed ball $ {x\in \Bbb{R}^{3} ; || x || \le 1} $ I want to draw Eulidean norm, $L_1 $ norm and $L_{\infty}$ norm by using matlab.

0
Q: Lagrange's theorem proof

NesaI read the answers to these questions: Proof of Lagrange theorem - Order of a subgroup divides order of the group Lagrange's theorem How do I know that the cosets of H have the same number of elements?(That equivalence classes form a partition of G.)

Short title. Short question. Lagrange's theorem proof
 
2:56 PM
0
Q: Should the XAMPP tag be removed?

DanielShould the xampp tag be removed? I can flag a question as off-topic with the following reason: Questions on Server Fault must be about managing information technology systems in a business environment. […] Questions about […] development tools may be asked on Stack Overflow. The toolset xa...

 
0
Q: Lebesgue measure on R

Al jabraI have a set of descriptions about the Lebesgue measure on $\mathbb{R}$. (i) A half open interval in $\mathbb{R}$ is a set of the form $[a,b[=\{x:a \leq x<b\}$ where $a,b \in\mathbb{R}$ We allow $b \leq a$ and in this case why does $[a,b[=\phi$? (ii) If $I \subseteq \mathbb{R}$ is a half open ...

 
3:11 PM
5
Q: Greeter Hat seems to be causing problems

apaul34208Is it just me or is the Greeter Hat causing a lot of questionable upvoting and trivial editing? Perhaps I'm growing cynical, but it seems like I've come across a lot of polished turds with single upvotes this morning. Greeter edit and upvote another user’s first post, which must have bee...

0
Q: Floating menu bar

ManikiranThe menu bar is stuck to the head, so its really hard for us to check whats happening while answering or when scrolled to the bottom. Making the menu bar floating can help us a lot, so that we can see it wherever we are on that page.

0
Q: Cannot add Meta Stack Overflow in my sites

rekireI cannot add Meta Stack Overflow on in the list of my sites on my phone in the Android App. Also when I got it managed to ask a question I get the feedback that my question was deleted. Is the app not meta ready?

 
0
Q: How to reduce this boolean expression (A+C+D)(A+C+D`)(A+C`+D)(A+B`) into four literals

user3305573I am try to simplifying this boolean expression (A+C+D)(A+C+D)(A+C+D)(A+B)`into four literals since from four days. Can anybody please help me out.

0
Q: Any continous function on $\mathbb R$ such that $f(n) = 0 $ for all $n \in \mathbb Z$

user120386Any continous function on $\mathbb R$ such that $f(n) = 0 $ for all $n \in \mathbb Z$ . Then Which one of the following is true 1) Image of $f$ is closed. 2) Image of $f$ is open. 3) $f$ is uniformly continous. I am unable to solve ,Please give me hint how to solve. Thank you

0
Q: Prove that $/gcd{a,b} = /gcd{b,a}$

GeoffI cant get my head round how to prove this, I tried using the euclidean algorithm but that didn't help me much.

 
3:31 PM
0
Q: What questions about computer building are on topic here?

HellreaverRecently, I proposed a stackexhange site for users to come and ask questions about very specific questions about computer building. (would it be safe to mount my motherboard to wood, etc). The proposal was immediately turned down, labeled a duplicate of superuser. Is it? What type of computer bui...

0
Q: How to deal with off-topic questions that cannot be closed?

kasperdA came across a question which looks off-topic to me. I voted to migrate the question, but got an error message saying: This question has an open bounty and cannot be closed How should we deal with such questions?

 
0
Q: kernel of action.

HawkIf $\phi : G \to Perm(G/H)$ where $\phi$ is the group action on $G/H$ by $G$. $\phi := g(g'H) = (gg')H$ Why is the kernel of $\phi$ equal to $\cap_{x\in G} xHx^{-1}$ I thought the kernel is $\ker \phi = \{g | gx =x \}$ (so ker is a stabilizer).So in our case it is $g(g'H) = g'H$ . So all forms...

Short title. kernel of action.
0
Q: Irrational numbers are limits of a rational sequence

VrouvrouPlease how to prove by using two methods that Irrational numbers are limits of a rational sequence After that deduce that $\mathbb{Q}$ is not closed in $(\mathbb{R},|.|)$ Thank you

0
Q: Proof by contrapositive: $ 4 \nmid (n-2)^2 \Rightarrow 6 \nmid n $

user108410Prove that if an integer n is not divisible by 6, then $ (n-2)^2 $ is not divisible by 4. Prove: $ 4 \nmid (n-2)^2 \Rightarrow 6 \nmid n $ Proof by contrapositive: $ 6 \mid n \Rightarrow 4 \mid (n-2)^2 $ n=6k,$ \\\ k \in \Bbb Z $ $((6k)-2)^2 = 36k^2 - 24k+4 = 4(9k^2 + 6k+1), (n-2)^2=4c \\\ w...

 
3:51 PM
0
Q: Is there a way to flag "related" items as not actually related?

devyndraenI came across a question about how to properly apply indexes in Oracle, and in the related column, there is a question about the GCC compiler's handling of the pow() function. Apart from the fact that both Oracle and GCC deal with floating point numbers and both questions have the optimization f...

 
0
Q: Non-standard version of Frechet derivative

PietzNon-standard analysis offers very convenient tools to prove facts about continuity or differentiability. I am looking for such tool in infinite-dimensional calculus. To be more precise, let $X$ and $Y$ be Banach spaces and let ${}^*X, {}^*Y$ be their non-standard extensions. Suppose we have fun...

Consider adding a tag for a broader subject area to which the question belongs. Some of these tags might fit. (autocomment)Normal Human 20 secs ago
0
Q: Intermediate galois fields

Makoto K.I want to find two different fields $K_1,K_2$ such that $\Bbb Q\subset K_i \subset \Bbb Q(\alpha,\zeta)$ such that $K_i /\Bbb Q$ are Galois. A few things: $\alpha$ is the real $6^{th}$ roof of $2$, $\zeta$ is a primitive $6^{th}$ root of unity. Let $L= \Bbb Q(\alpha,\zeta)$, I have shown here ...

 
00:00 - 16:0016:00 - 00:00

« first day (42 days earlier)      last day (532 days later) »